1987 AIME Problems/Problem 9

Revision as of 00:52, 11 February 2007 by Minsoens (talk | contribs)

Problem

Triangle $\displaystyle ABC$ has right angle at $\displaystyle B$, and contains a point $\displaystyle P$ for which $\displaystyle PA = 10$, $\displaystyle PB = 6$, and $\displaystyle \angle APB = \angle BPC = \angle CPA$. Find $\displaystyle PC$.

AIME 1987 Problem 9.png

Solution

This problem needs a solution. If you have a solution for it, please help us out by adding it.

See also

1987 AIME (ProblemsAnswer KeyResources)
Preceded by
Problem 8
Followed by
Problem 10
1 2 3 4 5 6 7 8 9 10 11 12 13 14 15
All AIME Problems and Solutions